¿Cómo ver que E⋅BE⋅B\mathbf{E}\cdot\mathbf{B} es una derivada total?

Desde mi B es un invariante de Lorentz de los campos electromagnéticos, parece interesante conectarlo a un Lagrangiano para ver qué sucede. Sin embargo, esto termina desapareciendo, y me dicen que esto debería ser obvio porque es un derivado total.

Sin embargo, esto no es obvio para mí. ¿Hay una manera fácil de ver eso?

1 2 ϵ α β γ d F α β F γ d = 4 C ( B mi )

es en realidad una derivada total?

También agradecería si alguien puede mostrar de qué es la derivada, de modo que pueda calcular la derivada para ayudar a asimilarla.

Si conoces las formas diferenciales, mi B Se puede escribir como F F . Entonces puedes encontrar fácilmente que se puede escribir como d ( A F ) dónde F = d A . Esta es exactamente la misma explicación que la de Javier.
@Minkyu No estoy familiarizado con las formas diferenciales, así que no lo vi como F F , pero con F = d A y la identidad d F = d d A = 0 , el resultado es bastante sencillo. Ahora puedo ver por qué sería obvio para aquellos que dominan las formas diferenciales. ¡Gracias!
Debo decir que lo que escribí anteriormente no es exactamente lo mismo que la respuesta de Javier después de darme cuenta del problema mencionado por Sean E. Lake. Pero mi comentario sigue siendo válido después de corregir el calibre mencionado por Sean E. Lake.
@Minkyu No puedo comentar sobre geometría diferencial; no estoy familiarizado con eso (todavía). Puedo decir que la publicación de Javier hace que la fijación de calibre sea innecesaria, simplemente conveniente. Es decir, muestra que la parte del potencial escalar se desvanece, y eso aún funciona incluso fuera del indicador de Weyl.
@Minkyu, la mejor razón para no poner este término en un Lagrangiano no es porque sea una derivada total, sino porque es un psuedoescalar, por lo que su acción 1) si contiene solo este término no es invariante bajo simetría transformación ( X X ), pero cambia solo su signo 2) si contiene otros términos que son escalares reales, no tiene propiedades de transformación claras bajo simetría y puede cambiar mucho
@Minkyu ¿Puedes publicar tu comentario como respuesta para que pueda aceptarlo? Realmente se siente como la mejor respuesta. Es tan breve y dulce que realmente capta por qué esto sería "obvio" para algunas personas, y explica la "magia" detrás de lo que sucede en las manipulaciones de las otras respuestas.
@PPenguin Acabo de publicar una respuesta. Espero que te ayude.

Respuestas (3)

Para completar, aquí está la versión de notación tensorial.

Primera reescritura:

( mi B )     ϵ α β γ d F α β F γ d = ϵ α β γ d ( α A β β A α ) F γ d = 2   ϵ α β γ d ( α A β ) F γ d

donde el último paso utiliza el reetiquetado y la antisimetría de ϵ α β γ d .

Similarmente

ϵ α β γ d ( α A β ) F γ d = 2   ϵ α β γ d ( α A β ) ( γ A d ) .

Ahora moviendo uno de los derivados al frente

ϵ α β γ d ( α A β ) ( γ A d ) = α ( ϵ α β γ d A β ( γ A d ) ) ϵ α β γ d A β ( α γ A d )

y tenga en cuenta que el último término es cero porque las derivadas conmutan y, por lo tanto, son simétricas en esas etiquetas, mientras que ϵ α β γ d es antisimétrico.

En total esto da:

( mi B )     α ( ϵ α β γ d A β ( γ A d ) )

Primera nota que puedes reescribir mi B como

mi B F F
usando una fuerza de campo 2 -forma F dónde mi y B se definen como
F 0 i = mi i , F i j = ϵ i j k B k .
Más específicamente,
F F = 1 4 F m v F ρ σ d X m d X v d X ρ d X σ = 1 4 ϵ m v ρ σ F m v F ρ σ vol. ϵ i j k F 0 i F j k = mi B .
Entonces es fácil demostrar que mi B es una derivada total usando F = d A , es decir,
mi B F F = d ( A F ) .


Como comentario al margen, F F contiene forma de volumen pero está ausente en mi B . Entonces la forma correcta de escribir es

d 4 X mi B F F .

La forma más fácil de examinar mi B es mirar los campos en términos del vector potencial en el calibre de Weyl , donde A 0 = 0 . En ese calibre, obtienes:

mi = A t ,   a norte d B = × A .

Así obtienes:

mi B = ϵ i j k A i t ( A k X j )
Ahora examinemos la integral de espacio-tiempo de esta cantidad:
mi B d X 4 = ϵ i j k A i t ( A k X j ) d X 4 = [ t ( ϵ i j k A i A k X j ) + ϵ i j k A i 2 A k X j t ] d X 4 i norte t mi gramo r a t mi   b y   pag a r t s   i norte   t = [ t ( ϵ i j k A i A k X j ) + X j ( ϵ i j k A i A k t ) ϵ i j k A k t ( A i X j ) ] d X 4 i norte t mi gramo r a t mi   b y   pag a r t s   i norte   s pag a C mi = ϵ k j i A i t ( A k X j ) d X 4 d r o pag   s tu r F a C mi   t mi r metro s a norte d   r mi yo a b mi yo   d tu metro metro y   v a r i a b yo mi s = ϵ i j k A i t ( A k X j ) d X 4 mi X C h a norte gramo mi   i norte d i C mi s   o F   ϵ
Observe que acabamos de demostrar que la integral es igual a menos ella misma y, por lo tanto, debe desaparecer si los términos de la superficie desaparecen.

+1 Nunca antes había oído hablar del indicador Weyl; ¡ahora sé!